Is there a closed form for the integral $int_0^infty frac{e^{-x^2} I_0 left(beta x right) d x}{sqrt{...












6














I encountered this integral in my work, and it would be really convenient if it had a closed form in terms of any known special functions (which Mathematica could handle):



$$J(alpha,beta)=int_0^infty frac{e^{-x^2} I_0 left(beta x right) d x}{sqrt{ alpha^2+x^2}}$$



It's easy to see that:



$$J(alpha,0)=frac12 e^{alpha^2/2} K_0 left( frac{alpha^2}{2} right)$$



Using series expansion of the Bessel function and a little help from Mathematica, I was able to get a series expression:



$$J(alpha,beta)=frac12 e^{alpha^2/2} K_0 left( frac{alpha^2}{2} right)+frac{sqrt{pi}}{16} beta^2 sum_{k=0}^infty frac{(2k+1)!}{k!^3 (k+1)^2} U left(frac{1}{2},-k,alpha^2 right) frac{beta^{2k}}{4^{2k}}$$



Where $U$ is the confluent hypergeometric function, which in this case (according to Mathematica) is a linear combination of Bessel functions $K_0$ and $K_1$ though I wasn't able to get the general expression for the coefficients yet.



This series is good for small values of $beta$, but for large $beta$ I need too many terms, and the computation time is slower than I'd like.



I know that it's simple to get a quadrature formula for the integral, but still, a closed form would be better.





The integral can be thought as a Hankel transform with imaginary argument, but I wasn't able to find it in the tables of Hankel transforms either.





Clarification re: R. Burton's comment:



The final expression has the form:



$$alpha e^{-beta^2/2} J(alpha,beta)$$



Which should make the expression finite for every choice of variables.





Now that I think about it, I should probably use asymptotics for the Bessel function for large $beta$:



$$I_0 left(beta x right) asymp frac{1}{sqrt{2 pi beta x}}e^{beta x}$$



Then I get:



$$J(alpha,beta) asymp frac{1}{sqrt{2 pi beta}} int_0^infty frac{e^{-x^2+beta x} d x}{sqrt{x}sqrt{ alpha^2+x^2}}, quad beta to infty$$



Which I still don't know the closed form for.





A more simple integral which does have a closed form:



$$int_0^infty e^{-x^2} I_0 left(beta x right) d x= frac{sqrt{pi}}{2} e^{beta^2/8} I_0 left(frac{beta^2}{8} right)$$










share|cite|improve this question
























  • I don't know about a closed form, but you can use the definition of $$I_0(x)=sum_{n=0}^infty frac{(beta x)^{2n}}{4^nn!^2}$$ to get $$J(alpha,beta,x)=int_0^infty frac{e^{-x^2}}{leftVert(alpha,x)rightVert}sum_{n=0}^infty frac{(beta x)^{2n}}{4^nn!^2} dx$$ If you truncate the sum it should at least reduce the computation time.
    – R. Burton
    Dec 26 '18 at 16:39










  • Also, I tried running this through Scilab, and for very large $beta$ (ex 130+), I ended up with things like $xtimes10^{103}$, %inf, and error messages. Suffice to say that for sufficiently large $beta$, the output of $J$ becomes impractically large.
    – R. Burton
    Dec 26 '18 at 16:46












  • @R.Burton, re: your first comment: this is what I did to get the series
    – Yuriy S
    Dec 26 '18 at 17:15










  • @R.Burton, I posted an edit to answer your second observation. The point is that for large $beta$ there's an exponential factor before the integral which should make it small. I guess instead of the series we could derive some kind of asymptotic expression for that case
    – Yuriy S
    Dec 26 '18 at 18:07












  • Is this somehow related to probability distributions or wavefunctions?
    – R. Burton
    Dec 27 '18 at 22:57
















6














I encountered this integral in my work, and it would be really convenient if it had a closed form in terms of any known special functions (which Mathematica could handle):



$$J(alpha,beta)=int_0^infty frac{e^{-x^2} I_0 left(beta x right) d x}{sqrt{ alpha^2+x^2}}$$



It's easy to see that:



$$J(alpha,0)=frac12 e^{alpha^2/2} K_0 left( frac{alpha^2}{2} right)$$



Using series expansion of the Bessel function and a little help from Mathematica, I was able to get a series expression:



$$J(alpha,beta)=frac12 e^{alpha^2/2} K_0 left( frac{alpha^2}{2} right)+frac{sqrt{pi}}{16} beta^2 sum_{k=0}^infty frac{(2k+1)!}{k!^3 (k+1)^2} U left(frac{1}{2},-k,alpha^2 right) frac{beta^{2k}}{4^{2k}}$$



Where $U$ is the confluent hypergeometric function, which in this case (according to Mathematica) is a linear combination of Bessel functions $K_0$ and $K_1$ though I wasn't able to get the general expression for the coefficients yet.



This series is good for small values of $beta$, but for large $beta$ I need too many terms, and the computation time is slower than I'd like.



I know that it's simple to get a quadrature formula for the integral, but still, a closed form would be better.





The integral can be thought as a Hankel transform with imaginary argument, but I wasn't able to find it in the tables of Hankel transforms either.





Clarification re: R. Burton's comment:



The final expression has the form:



$$alpha e^{-beta^2/2} J(alpha,beta)$$



Which should make the expression finite for every choice of variables.





Now that I think about it, I should probably use asymptotics for the Bessel function for large $beta$:



$$I_0 left(beta x right) asymp frac{1}{sqrt{2 pi beta x}}e^{beta x}$$



Then I get:



$$J(alpha,beta) asymp frac{1}{sqrt{2 pi beta}} int_0^infty frac{e^{-x^2+beta x} d x}{sqrt{x}sqrt{ alpha^2+x^2}}, quad beta to infty$$



Which I still don't know the closed form for.





A more simple integral which does have a closed form:



$$int_0^infty e^{-x^2} I_0 left(beta x right) d x= frac{sqrt{pi}}{2} e^{beta^2/8} I_0 left(frac{beta^2}{8} right)$$










share|cite|improve this question
























  • I don't know about a closed form, but you can use the definition of $$I_0(x)=sum_{n=0}^infty frac{(beta x)^{2n}}{4^nn!^2}$$ to get $$J(alpha,beta,x)=int_0^infty frac{e^{-x^2}}{leftVert(alpha,x)rightVert}sum_{n=0}^infty frac{(beta x)^{2n}}{4^nn!^2} dx$$ If you truncate the sum it should at least reduce the computation time.
    – R. Burton
    Dec 26 '18 at 16:39










  • Also, I tried running this through Scilab, and for very large $beta$ (ex 130+), I ended up with things like $xtimes10^{103}$, %inf, and error messages. Suffice to say that for sufficiently large $beta$, the output of $J$ becomes impractically large.
    – R. Burton
    Dec 26 '18 at 16:46












  • @R.Burton, re: your first comment: this is what I did to get the series
    – Yuriy S
    Dec 26 '18 at 17:15










  • @R.Burton, I posted an edit to answer your second observation. The point is that for large $beta$ there's an exponential factor before the integral which should make it small. I guess instead of the series we could derive some kind of asymptotic expression for that case
    – Yuriy S
    Dec 26 '18 at 18:07












  • Is this somehow related to probability distributions or wavefunctions?
    – R. Burton
    Dec 27 '18 at 22:57














6












6








6


1





I encountered this integral in my work, and it would be really convenient if it had a closed form in terms of any known special functions (which Mathematica could handle):



$$J(alpha,beta)=int_0^infty frac{e^{-x^2} I_0 left(beta x right) d x}{sqrt{ alpha^2+x^2}}$$



It's easy to see that:



$$J(alpha,0)=frac12 e^{alpha^2/2} K_0 left( frac{alpha^2}{2} right)$$



Using series expansion of the Bessel function and a little help from Mathematica, I was able to get a series expression:



$$J(alpha,beta)=frac12 e^{alpha^2/2} K_0 left( frac{alpha^2}{2} right)+frac{sqrt{pi}}{16} beta^2 sum_{k=0}^infty frac{(2k+1)!}{k!^3 (k+1)^2} U left(frac{1}{2},-k,alpha^2 right) frac{beta^{2k}}{4^{2k}}$$



Where $U$ is the confluent hypergeometric function, which in this case (according to Mathematica) is a linear combination of Bessel functions $K_0$ and $K_1$ though I wasn't able to get the general expression for the coefficients yet.



This series is good for small values of $beta$, but for large $beta$ I need too many terms, and the computation time is slower than I'd like.



I know that it's simple to get a quadrature formula for the integral, but still, a closed form would be better.





The integral can be thought as a Hankel transform with imaginary argument, but I wasn't able to find it in the tables of Hankel transforms either.





Clarification re: R. Burton's comment:



The final expression has the form:



$$alpha e^{-beta^2/2} J(alpha,beta)$$



Which should make the expression finite for every choice of variables.





Now that I think about it, I should probably use asymptotics for the Bessel function for large $beta$:



$$I_0 left(beta x right) asymp frac{1}{sqrt{2 pi beta x}}e^{beta x}$$



Then I get:



$$J(alpha,beta) asymp frac{1}{sqrt{2 pi beta}} int_0^infty frac{e^{-x^2+beta x} d x}{sqrt{x}sqrt{ alpha^2+x^2}}, quad beta to infty$$



Which I still don't know the closed form for.





A more simple integral which does have a closed form:



$$int_0^infty e^{-x^2} I_0 left(beta x right) d x= frac{sqrt{pi}}{2} e^{beta^2/8} I_0 left(frac{beta^2}{8} right)$$










share|cite|improve this question















I encountered this integral in my work, and it would be really convenient if it had a closed form in terms of any known special functions (which Mathematica could handle):



$$J(alpha,beta)=int_0^infty frac{e^{-x^2} I_0 left(beta x right) d x}{sqrt{ alpha^2+x^2}}$$



It's easy to see that:



$$J(alpha,0)=frac12 e^{alpha^2/2} K_0 left( frac{alpha^2}{2} right)$$



Using series expansion of the Bessel function and a little help from Mathematica, I was able to get a series expression:



$$J(alpha,beta)=frac12 e^{alpha^2/2} K_0 left( frac{alpha^2}{2} right)+frac{sqrt{pi}}{16} beta^2 sum_{k=0}^infty frac{(2k+1)!}{k!^3 (k+1)^2} U left(frac{1}{2},-k,alpha^2 right) frac{beta^{2k}}{4^{2k}}$$



Where $U$ is the confluent hypergeometric function, which in this case (according to Mathematica) is a linear combination of Bessel functions $K_0$ and $K_1$ though I wasn't able to get the general expression for the coefficients yet.



This series is good for small values of $beta$, but for large $beta$ I need too many terms, and the computation time is slower than I'd like.



I know that it's simple to get a quadrature formula for the integral, but still, a closed form would be better.





The integral can be thought as a Hankel transform with imaginary argument, but I wasn't able to find it in the tables of Hankel transforms either.





Clarification re: R. Burton's comment:



The final expression has the form:



$$alpha e^{-beta^2/2} J(alpha,beta)$$



Which should make the expression finite for every choice of variables.





Now that I think about it, I should probably use asymptotics for the Bessel function for large $beta$:



$$I_0 left(beta x right) asymp frac{1}{sqrt{2 pi beta x}}e^{beta x}$$



Then I get:



$$J(alpha,beta) asymp frac{1}{sqrt{2 pi beta}} int_0^infty frac{e^{-x^2+beta x} d x}{sqrt{x}sqrt{ alpha^2+x^2}}, quad beta to infty$$



Which I still don't know the closed form for.





A more simple integral which does have a closed form:



$$int_0^infty e^{-x^2} I_0 left(beta x right) d x= frac{sqrt{pi}}{2} e^{beta^2/8} I_0 left(frac{beta^2}{8} right)$$







definite-integrals closed-form bessel-functions






share|cite|improve this question















share|cite|improve this question













share|cite|improve this question




share|cite|improve this question








edited Dec 27 '18 at 11:08

























asked Dec 26 '18 at 11:41









Yuriy S

15.7k433117




15.7k433117












  • I don't know about a closed form, but you can use the definition of $$I_0(x)=sum_{n=0}^infty frac{(beta x)^{2n}}{4^nn!^2}$$ to get $$J(alpha,beta,x)=int_0^infty frac{e^{-x^2}}{leftVert(alpha,x)rightVert}sum_{n=0}^infty frac{(beta x)^{2n}}{4^nn!^2} dx$$ If you truncate the sum it should at least reduce the computation time.
    – R. Burton
    Dec 26 '18 at 16:39










  • Also, I tried running this through Scilab, and for very large $beta$ (ex 130+), I ended up with things like $xtimes10^{103}$, %inf, and error messages. Suffice to say that for sufficiently large $beta$, the output of $J$ becomes impractically large.
    – R. Burton
    Dec 26 '18 at 16:46












  • @R.Burton, re: your first comment: this is what I did to get the series
    – Yuriy S
    Dec 26 '18 at 17:15










  • @R.Burton, I posted an edit to answer your second observation. The point is that for large $beta$ there's an exponential factor before the integral which should make it small. I guess instead of the series we could derive some kind of asymptotic expression for that case
    – Yuriy S
    Dec 26 '18 at 18:07












  • Is this somehow related to probability distributions or wavefunctions?
    – R. Burton
    Dec 27 '18 at 22:57


















  • I don't know about a closed form, but you can use the definition of $$I_0(x)=sum_{n=0}^infty frac{(beta x)^{2n}}{4^nn!^2}$$ to get $$J(alpha,beta,x)=int_0^infty frac{e^{-x^2}}{leftVert(alpha,x)rightVert}sum_{n=0}^infty frac{(beta x)^{2n}}{4^nn!^2} dx$$ If you truncate the sum it should at least reduce the computation time.
    – R. Burton
    Dec 26 '18 at 16:39










  • Also, I tried running this through Scilab, and for very large $beta$ (ex 130+), I ended up with things like $xtimes10^{103}$, %inf, and error messages. Suffice to say that for sufficiently large $beta$, the output of $J$ becomes impractically large.
    – R. Burton
    Dec 26 '18 at 16:46












  • @R.Burton, re: your first comment: this is what I did to get the series
    – Yuriy S
    Dec 26 '18 at 17:15










  • @R.Burton, I posted an edit to answer your second observation. The point is that for large $beta$ there's an exponential factor before the integral which should make it small. I guess instead of the series we could derive some kind of asymptotic expression for that case
    – Yuriy S
    Dec 26 '18 at 18:07












  • Is this somehow related to probability distributions or wavefunctions?
    – R. Burton
    Dec 27 '18 at 22:57
















I don't know about a closed form, but you can use the definition of $$I_0(x)=sum_{n=0}^infty frac{(beta x)^{2n}}{4^nn!^2}$$ to get $$J(alpha,beta,x)=int_0^infty frac{e^{-x^2}}{leftVert(alpha,x)rightVert}sum_{n=0}^infty frac{(beta x)^{2n}}{4^nn!^2} dx$$ If you truncate the sum it should at least reduce the computation time.
– R. Burton
Dec 26 '18 at 16:39




I don't know about a closed form, but you can use the definition of $$I_0(x)=sum_{n=0}^infty frac{(beta x)^{2n}}{4^nn!^2}$$ to get $$J(alpha,beta,x)=int_0^infty frac{e^{-x^2}}{leftVert(alpha,x)rightVert}sum_{n=0}^infty frac{(beta x)^{2n}}{4^nn!^2} dx$$ If you truncate the sum it should at least reduce the computation time.
– R. Burton
Dec 26 '18 at 16:39












Also, I tried running this through Scilab, and for very large $beta$ (ex 130+), I ended up with things like $xtimes10^{103}$, %inf, and error messages. Suffice to say that for sufficiently large $beta$, the output of $J$ becomes impractically large.
– R. Burton
Dec 26 '18 at 16:46






Also, I tried running this through Scilab, and for very large $beta$ (ex 130+), I ended up with things like $xtimes10^{103}$, %inf, and error messages. Suffice to say that for sufficiently large $beta$, the output of $J$ becomes impractically large.
– R. Burton
Dec 26 '18 at 16:46














@R.Burton, re: your first comment: this is what I did to get the series
– Yuriy S
Dec 26 '18 at 17:15




@R.Burton, re: your first comment: this is what I did to get the series
– Yuriy S
Dec 26 '18 at 17:15












@R.Burton, I posted an edit to answer your second observation. The point is that for large $beta$ there's an exponential factor before the integral which should make it small. I guess instead of the series we could derive some kind of asymptotic expression for that case
– Yuriy S
Dec 26 '18 at 18:07






@R.Burton, I posted an edit to answer your second observation. The point is that for large $beta$ there's an exponential factor before the integral which should make it small. I guess instead of the series we could derive some kind of asymptotic expression for that case
– Yuriy S
Dec 26 '18 at 18:07














Is this somehow related to probability distributions or wavefunctions?
– R. Burton
Dec 27 '18 at 22:57




Is this somehow related to probability distributions or wavefunctions?
– R. Burton
Dec 27 '18 at 22:57










0






active

oldest

votes











Your Answer





StackExchange.ifUsing("editor", function () {
return StackExchange.using("mathjaxEditing", function () {
StackExchange.MarkdownEditor.creationCallbacks.add(function (editor, postfix) {
StackExchange.mathjaxEditing.prepareWmdForMathJax(editor, postfix, [["$", "$"], ["\\(","\\)"]]);
});
});
}, "mathjax-editing");

StackExchange.ready(function() {
var channelOptions = {
tags: "".split(" "),
id: "69"
};
initTagRenderer("".split(" "), "".split(" "), channelOptions);

StackExchange.using("externalEditor", function() {
// Have to fire editor after snippets, if snippets enabled
if (StackExchange.settings.snippets.snippetsEnabled) {
StackExchange.using("snippets", function() {
createEditor();
});
}
else {
createEditor();
}
});

function createEditor() {
StackExchange.prepareEditor({
heartbeatType: 'answer',
autoActivateHeartbeat: false,
convertImagesToLinks: true,
noModals: true,
showLowRepImageUploadWarning: true,
reputationToPostImages: 10,
bindNavPrevention: true,
postfix: "",
imageUploader: {
brandingHtml: "Powered by u003ca class="icon-imgur-white" href="https://imgur.com/"u003eu003c/au003e",
contentPolicyHtml: "User contributions licensed under u003ca href="https://creativecommons.org/licenses/by-sa/3.0/"u003ecc by-sa 3.0 with attribution requiredu003c/au003e u003ca href="https://stackoverflow.com/legal/content-policy"u003e(content policy)u003c/au003e",
allowUrls: true
},
noCode: true, onDemand: true,
discardSelector: ".discard-answer"
,immediatelyShowMarkdownHelp:true
});


}
});














draft saved

draft discarded


















StackExchange.ready(
function () {
StackExchange.openid.initPostLogin('.new-post-login', 'https%3a%2f%2fmath.stackexchange.com%2fquestions%2f3052862%2fis-there-a-closed-form-for-the-integral-int-0-infty-frace-x2-i-0-left%23new-answer', 'question_page');
}
);

Post as a guest















Required, but never shown

























0






active

oldest

votes








0






active

oldest

votes









active

oldest

votes






active

oldest

votes
















draft saved

draft discarded




















































Thanks for contributing an answer to Mathematics Stack Exchange!


  • Please be sure to answer the question. Provide details and share your research!

But avoid



  • Asking for help, clarification, or responding to other answers.

  • Making statements based on opinion; back them up with references or personal experience.


Use MathJax to format equations. MathJax reference.


To learn more, see our tips on writing great answers.





Some of your past answers have not been well-received, and you're in danger of being blocked from answering.


Please pay close attention to the following guidance:


  • Please be sure to answer the question. Provide details and share your research!

But avoid



  • Asking for help, clarification, or responding to other answers.

  • Making statements based on opinion; back them up with references or personal experience.


To learn more, see our tips on writing great answers.




draft saved


draft discarded














StackExchange.ready(
function () {
StackExchange.openid.initPostLogin('.new-post-login', 'https%3a%2f%2fmath.stackexchange.com%2fquestions%2f3052862%2fis-there-a-closed-form-for-the-integral-int-0-infty-frace-x2-i-0-left%23new-answer', 'question_page');
}
);

Post as a guest















Required, but never shown





















































Required, but never shown














Required, but never shown












Required, but never shown







Required, but never shown

































Required, but never shown














Required, but never shown












Required, but never shown







Required, but never shown







Popular posts from this blog

Human spaceflight

Can not write log (Is /dev/pts mounted?) - openpty in Ubuntu-on-Windows?

File:DeusFollowingSea.jpg